Difference between revisions of "2021 AMC 10B Problems/Problem 16"

m (Solution 4 (Casework on Deleting the Digits of 12345))
(new solution)
(One intermediate revision by one other user not shown)
Line 50: Line 50:
 
~coolmath34
 
~coolmath34
  
==Solution 4 (Casework on Deleting the Digits of 12345)==
+
==Solution 4==
For every positive integer:
+
An integer is divisible by <math>15</math> iff it is divisible by <math>3</math> and <math>5</math>. Divisibility by <math>5</math> means ending in <math>0</math> or <math>5</math>, but since no digit is less than <math>0</math>, the only uphill integer ending in <math>0</math> could be <math>0</math>, which is not positive. This means the integer must end in <math>5</math>.
  
* It is divisible by <math>3</math> if and only if its digit-sum is divisible by <math>3.</math>
+
All uphill integers ending in <math>5</math> are formed by picking any subset of the sequence <math>(1,2,3,4)</math> of digits (keeping their order), then appending a <math>5</math>. Divisibility by <math>3</math> means the sum of the digits is a multiple of <math>3</math>, so our choice of digits must add to <math>0</math> modulo <math>3</math>.
  
* It is divisible by <math>5</math> if and only if its units digit is <math>0</math> or <math>5.</math>  
+
<math>5 \equiv -1 \pmod{3}</math>, so the other digits we pick must add to <math>1</math> modulo <math>3</math>. Since <math>(1,2,3,4) \equiv (1,-1,0,1) \pmod{3}</math>, we can pick either nothing, or one residue <math>1</math> (from <math>1</math> or <math>4</math>) and one residue <math>-1</math> (from <math>2</math>), and we can then optionally add a residue <math>0</math> (from <math>3</math>). This gives <math>(1+2\cdot1)\cdot2 = \boxed{\textbf{(C)}~6}</math> possibilities.
  
* It is divisible by <math>15</math> if and only if it is divisible by both <math>3</math> and <math>5.</math>
+
~[[User:emerald_block|emerald_block]]
 
 
Since the desired positive integers are uphill, their units digits must be <math>5</math>s. We start with the largest such uphill integer (<math>12345,</math> by inspection), then perform casework on deleting its digits. <i><b>Clearly, we cannot delete the digit <math>\boldsymbol{5,}</math> as that is the only way to satisfy the divisibility rule of <math>\boldsymbol{5.}</math></b></i> Now, we focus on the divisibility rule of <math>3.</math>
 
 
 
Note that the sum of the deleted digits must be divisible by <math>3,</math> so the difference between <math>1+2+3+4+5=15</math> and this sum is also divisible by <math>3</math> (Quick Proof: Suppose the sum of the deleted digits is <math>3k.</math> It follows that <math>15-3k=3(5-k)</math> must be divisible by <math>3.</math>).
 
 
 
Two solutions follow from here:
 
 
 
===Solution 4.1 (Casework on the Number of Digits Deleted)===
 
<b>Case (1): Delete exactly <math>\boldsymbol{0}</math> digits. (<math>\boldsymbol{5}</math>-digit uphill integers)</b>
 
 
 
There is <math>1</math> uphill integer in this case: <math>12345.</math>
 
 
 
<b>Case (2): Delete exactly <math>\boldsymbol{1}</math> digit. (<math>\boldsymbol{4}</math>-digit uphill integers)</b>
 
 
 
We can only delete the digit <math>3.</math> So, there is <math>1</math> uphill integer in this case: <math>1245.</math>
 
 
 
<b>Case (3): Delete exactly <math>\boldsymbol{2}</math> digits. (<math>\boldsymbol{3}</math>-digit uphill integers)</b>
 
 
 
We can only delete the digits that sum to either <math>3</math> or <math>6.</math> So, there are <math>2</math> uphill integers in this case: <math>345,135.</math>
 
 
 
<b>Case (4): Delete exactly <math>\boldsymbol{3}</math> digits. (<math>\boldsymbol{2}</math>-digit uphill integers)</b>
 
 
 
We can only delete the digits that sum to either <math>6</math> or <math>9.</math> So, there are <math>2</math> uphill integers in this case: <math>45,15.</math>
 
 
 
<b>Total</b>
 
 
 
Together, the answer is <math>1+1+2+2=\boxed{\textbf{(C)} ~6}.</math>
 
 
 
~MRENTHUSIASM
 
 
 
===Solution 4.2 (Casework on the Sum of Digits Deleted)===
 
<b>Case (1): The deleted digits' sum is <math>\boldsymbol{0.}</math> (The remaining digits' sum is <math>\boldsymbol{15.}</math>)</b>
 
 
 
There is <math>1</math> uphill integer in this case: <math>12345.</math>
 
 
 
<b>Case (2): The deleted digits' sum is <math>\boldsymbol{3.}</math> (The remaining digits' sum is <math>\boldsymbol{12.}</math>)</b>
 
 
 
Note that <math>3=1+2.</math> So, there are <math>2</math> uphill integers in this case: <math>1245,345.</math>
 
 
 
<b>Case (3): The deleted digits' sum is <math>\boldsymbol{6.}</math> (The remaining digits' sum is <math>\boldsymbol{9.}</math>)</b>
 
 
 
Note that <math>6=2+4=1+2+3.</math> So, there are <math>2</math> uphill integers in this case: <math>135,45.</math>
 
 
 
<b>Case (4): The deleted digits' sum is <math>\boldsymbol{9.}</math> (The remaining digits' sum is <math>\boldsymbol{6.}</math>)</b>
 
 
 
Note that <math>9=2+3+4.</math> So, there is <math>1</math> uphill integer in this case: <math>15.</math>
 
 
 
<b>Total</b>
 
 
 
Together, the answer is <math>1+2+2+1=\boxed{\textbf{(C)} ~6}.</math>
 
 
 
~MRENTHUSIASM
 
  
 
== Video Solution by OmegaLearn (Using Divisibility Rules and Casework) ==
 
== Video Solution by OmegaLearn (Using Divisibility Rules and Casework) ==

Revision as of 16:43, 7 November 2021

Problem

Call a positive integer an uphill integer if every digit is strictly greater than the previous digit. For example, $1357, 89,$ and $5$ are all uphill integers, but $32, 1240,$ and $466$ are not. How many uphill integers are divisible by $15$?

$\textbf{(A)} ~4 \qquad\textbf{(B)} ~5 \qquad\textbf{(C)} ~6 \qquad\textbf{(D)} ~7 \qquad\textbf{(E)} ~8$

Solution 1

The divisibility rule of $15$ is that the number must be congruent to $0$ mod $3$ and congruent to $0$ mod $5$. Being divisible by $5$ means that it must end with a $5$ or a $0$. We can rule out the case when the number ends with a $0$ immediately because the only integer that is uphill and ends with a $0$ is $0$ which is not positive. So now we know that the number ends with a $5$. Looking at the answer choices, the answer choices are all pretty small, so we can generate all of the numbers that are uphill and are divisible by $3$. These numbers are $15, 45, 135, 345, 1245, 12345$, or $\boxed{\textbf{(C)} ~6}$ numbers.

~ilikemath40

Solution 2

First, note how the number must end in either $5$ or $0$ in order to satisfying being divisible by $15$. However, the number can't end in $0$ because it's not strictly greater than the previous digits. Thus, our number must end in $5$. We do casework on the number of digits.

Case 1: $1$ digit. No numbers work, so $0$ numbers.

Case 2: $2$ digits. We have the numbers $15, 45,$ and $75$, but $75$ isn't an uphill number, so $2$ numbers

Case 3: $3$ digits. We have the numbers $135, 345$, so $2$ numbers.

Case 4: $4$ digits. We have the numbers $1235, 1245$ and $2345$, but only $1245$ satisfies this condition, so $1$ number.

Case 5: $5$ digits. We have only $12345$, so $1$ number.

Adding these up, we have $2+2+1+1=\boxed{\textbf{(C)} ~6}$.

~JustinLee2017

Solution 3

Like solution 2, we can proceed by using casework. A number is divisible by $15$ if is divisible by $3$ and $5.$ In this case, the units digit must be $5,$ otherwise no number can be formed.

Case 1: sum of digits = 6

There is only one number, $15.$

Case 2: sum of digits = 9

There are two numbers: $45$ and $135.$

Case 3: sum of digits = 12

There are two numbers: $345$ and $1245.$

Case 4: sum of digits = 15

There is only one number, $12345.$

We can see that we have exhausted all cases, because in order to have a larger sum of digits, then a number greater than $5$ needs to be used, breaking the conditions of the problem. The answer is $\boxed{\textbf{(C)} ~6}.$

~coolmath34

Solution 4

An integer is divisible by $15$ iff it is divisible by $3$ and $5$. Divisibility by $5$ means ending in $0$ or $5$, but since no digit is less than $0$, the only uphill integer ending in $0$ could be $0$, which is not positive. This means the integer must end in $5$.

All uphill integers ending in $5$ are formed by picking any subset of the sequence $(1,2,3,4)$ of digits (keeping their order), then appending a $5$. Divisibility by $3$ means the sum of the digits is a multiple of $3$, so our choice of digits must add to $0$ modulo $3$.

$5 \equiv -1 \pmod{3}$, so the other digits we pick must add to $1$ modulo $3$. Since $(1,2,3,4) \equiv (1,-1,0,1) \pmod{3}$, we can pick either nothing, or one residue $1$ (from $1$ or $4$) and one residue $-1$ (from $2$), and we can then optionally add a residue $0$ (from $3$). This gives $(1+2\cdot1)\cdot2 = \boxed{\textbf{(C)}~6}$ possibilities.

~emerald_block

Video Solution by OmegaLearn (Using Divisibility Rules and Casework)

https://youtu.be/n2FnKxFSW94

~ pi_is_3.14

Video Solution by TheBeautyofMath

https://youtu.be/FV9AnyERgJQ

~IceMatrix

Video Solution by Interstigation

https://youtu.be/9ZlJTVhtu_s

~Interstigation

See Also

2021 AMC 10B (ProblemsAnswer KeyResources)
Preceded by
Problem 15
Followed by
Problem 17
1 2 3 4 5 6 7 8 9 10 11 12 13 14 15 16 17 18 19 20 21 22 23 24 25
All AMC 10 Problems and Solutions

The problems on this page are copyrighted by the Mathematical Association of America's American Mathematics Competitions. AMC logo.png